DS Prob from GMAT Prep

This topic has expert replies
Senior | Next Rank: 100 Posts
Posts: 51
Joined: Wed Mar 09, 2011 2:34 am
Thanked: 2 times

DS Prob from GMAT Prep

by nishant1309 » Sun Mar 13, 2011 12:58 pm
Henry purchased 3 items during a sale. He received 20% discount off regular price of the most expensive item and a 10% discount off regular price off each of the other 2 items. Was the total amount of discount greater than 15% of the sum of the regular prices of all the 3 items?

1. Regular price for most expensive item: $50 & regular price of the next expensive item: $ 20.

2. Regular price for least expensive item: $ 15.

Legendary Member
Posts: 1337
Joined: Sat Dec 27, 2008 6:29 pm
Thanked: 127 times
Followed by:10 members

by Night reader » Sun Mar 13, 2011 1:09 pm
let us call the most exp. item A, the other two items B and C. We need to find if (0.8A+0.1B+0.1C)/(A+B+C)>0.85
st(1) A=50 and B=C=20 --> (40+18+18)/90<0.85 hence we can answer No on this question, Sufficient;
st(2) we have only C or B=15 BUT A can be anything, hence Not Sufficient

answer A
nishant1309 wrote:Henry purchased 3 items during a sale. He received 20% discount off regular price of the most expensive item and a 10% discount off regular price off each of the other 2 items. Was the total amount of discount greater than 15% of the sum of the regular prices of all the 3 items?

1. Regular price for most expensive item: $50 & regular price of the next expensive item: $ 20.

2. Regular price for least expensive item: $ 15.
My knowledge frontiers came to evolve the GMATPill's methods - the credited study means to boost the Verbal competence. I really like their videos, especially for RC, CR and SC. You do check their study methods at https://www.gmatpill.com

Senior | Next Rank: 100 Posts
Posts: 51
Joined: Wed Mar 09, 2011 2:34 am
Thanked: 2 times

by nishant1309 » Sun Mar 13, 2011 1:56 pm
Night reader wrote:let us call the most exp. item A, the other two items B and C. We need to find if (0.8A+0.1B+0.1C)/(A+B+C)>0.85
st(1) A=50 and B=C=20 --> (40+18+18)/90<0.85 hence we can answer No on this question, Sufficient;
st(2) we have only C or B=15 BUT A can be anything, hence Not Sufficient

answer A
nishant1309 wrote:Henry purchased 3 items during a sale. He received 20% discount off regular price of the most expensive item and a 10% discount off regular price off each of the other 2 items. Was the total amount of discount greater than 15% of the sum of the regular prices of all the 3 items?

1. Regular price for most expensive item: $50 & regular price of the next expensive item: $ 20.

2. Regular price for least expensive item: $ 15.
@ Night rider: To my understanding:
1. we need to check whether (0.2A+0.1B+0.1C)/(A+B+C) > 0.15.
2. St. 1 doesn't specifies regular price of least expensive item. You have assumed it to be = $ 20.

Please correct me if i got the question wrong.

Legendary Member
Posts: 1337
Joined: Sat Dec 27, 2008 6:29 pm
Thanked: 127 times
Followed by:10 members

by Night reader » Sun Mar 13, 2011 2:08 pm
look I have not assumed anything -- you are given three items // correct?
one is most expensive with discount 20% // correct?
and the other two less expensive with discount 10% // correct?
in st(1) the most expensive item is priced at $50 and the next expensive is priced at $20// correct?
from st(1) you can deduce that the next after next OR just one which is not most expensive could be at MOST $20 //correct?

hence you take A=50, B=C=20 // correct?

if you answer on every query "correct?" yes, please have your doubt resolved.
nishant1309 wrote:
Night reader wrote:let us call the most exp. item A, the other two items B and C. We need to find if (0.8A+0.1B+0.1C)/(A+B+C)>0.85
st(1) A=50 and B=C=20 --> (40+18+18)/90<0.85 hence we can answer No on this question, Sufficient;
st(2) we have only C or B=15 BUT A can be anything, hence Not Sufficient

answer A
nishant1309 wrote:Henry purchased 3 items during a sale. He received 20% discount off regular price of the most expensive item and a 10% discount off regular price off each of the other 2 items. Was the total amount of discount greater than 15% of the sum of the regular prices of all the 3 items?

1. Regular price for most expensive item: $50 & regular price of the next expensive item: $ 20.

2. Regular price for least expensive item: $ 15.
@ Night rider: To my understanding:
1. we need to check whether (0.2A+0.1B+0.1C)/(A+B+C) > 0.15.
2. St. 1 doesn't specifies regular price of least expensive item. You have assumed it to be = $ 20.

Please correct me if i got the question wrong.
My knowledge frontiers came to evolve the GMATPill's methods - the credited study means to boost the Verbal competence. I really like their videos, especially for RC, CR and SC. You do check their study methods at https://www.gmatpill.com

User avatar
GMAT Instructor
Posts: 15539
Joined: Tue May 25, 2010 12:04 pm
Location: New York, NY
Thanked: 13060 times
Followed by:1906 members
GMAT Score:790

by GMATGuruNY » Sun Mar 13, 2011 3:20 pm

Henry purchase 3 items during a sale. He received a 20 percent discount off the regular price of the most
expensive item and a 10 percent discount off the regular price of each of the other 2 items. Was the total
amount of the 3 discounts greater than 15 percent of the sum of the regular prices of the 3 items?
(1) The regular price of the most expensive item was $50, and the regular price of the next most expensive
item was $20
(2) The regular price of the least expensive item was $15.
This is a weighted average question.
How can we combine a 20% discount with a 10% discount to yield a combined discount that is more than 15%?
If the price discounted at 20% equals the sum of the prices discounted at 10%, the resulting discount will be exactly 15%.
Thus, to yield a discount that is more than 15%, the price discounted at 20% must be greater than the sum of the prices discounted at 10%.

Rephrased, the question is asking:

Is the regular price of the most expensive item greater than the sum of the regular prices of the two cheaper items?

Statement 1: The regular price of the most expensive item was $50, and the regular price of the next most expensive
item was $20.

The sum of the prices of the 2 cheaper items cannot be greater than 20+20 = 40.
Thus, the $50 price of the most expensive item must be greater than the sum of the prices of the 2 cheaper items.
Sufficient.

Statement 2: The regular price of the least expensive item was $15.
No way to determine whether the price of the most expensive item is greater than the sum of the prices of the 2 cheaper items.
Insufficient.

The correct answer is A.
Private tutor exclusively for the GMAT and GRE, with over 20 years of experience.
Followed here and elsewhere by over 1900 test-takers.
I have worked with students based in the US, Australia, Taiwan, China, Tajikistan, Kuwait, Saudi Arabia -- a long list of countries.
My students have been admitted to HBS, CBS, Tuck, Yale, Stern, Fuqua -- a long list of top programs.

As a tutor, I don't simply teach you how I would approach problems.
I unlock the best way for YOU to solve problems.

For more information, please email me (Mitch Hunt) at [email protected].
Student Review #1
Student Review #2
Student Review #3

User avatar
Senior | Next Rank: 100 Posts
Posts: 78
Joined: Fri Jul 23, 2010 2:22 am
Thanked: 2 times
Followed by:1 members

by gig92 » Mon Mar 14, 2011 5:42 am
nishant1309 wrote:Henry purchased 3 items during a sale. He received 20% discount off regular price of the most expensive item and a 10% discount off regular price off each of the other 2 items. Was the total amount of discount greater than 15% of the sum of the regular prices of all the 3 items?

1. Regular price for most expensive item: $50 & regular price of the next expensive item: $ 20.

2. Regular price for least expensive item: $ 15.
One more :

Stmnt 2: Regular price for least expensive item: $ 15

It doesn't give any idea about prices of other two items.It could be $1000 each or $2000 and $1000
So not Sufficient

Stmnt1:

We know from the discount of 20% on $50 = $10 and the discount of 10% on $20 = $2
So to answer: Was the total amount of discount greater than 15% of the sum of the regular prices of all the 3 items?
We can imagine that third item is of min value of $1 and max of $20 (cause the third item can have the same price as the second one). Now 10% discount of $1 to $20 will go to a max of $2. (ONE can easily imagine that the third item can also be of $20 as there are no restrictions on the price).

The question is: Was the total amount of discount greater than 15% of the sum of the regular prices of all the 3 items?

AND from the information in Stmnt 1, one can always find a unique answer.
Hence [spoiler]A[/spoiler]


NB: This is more of a "all in the head" approach!! Mitch has explained it better with the whole concept. However, sometimes one might not have the bells ringing to see that! So a faster idea in the head may tell you that Stmnt 1 gives enough information to anwser the question.
gig92